RegistrierenRegistrieren   LoginLogin   FAQFAQ    SuchenSuchen   
Stoß zweier Kugeln
 
Neue Frage »
Antworten »
    Foren-Übersicht -> Mechanik
Autor Nachricht
derphysiker40



Anmeldungsdatum: 28.10.2019
Beiträge: 50

Beitrag derphysiker40 Verfasst am: 26. Nov 2019 00:41    Titel: Stoß zweier Kugeln Antworten mit Zitat

Meine Frage:
Nabend,
ich habe schon heute paar Aufgaben zu den Stoßgesetzen gemacht, die für mich auch relativ leicht waren und schnell erledigt waren. Jedoch habe ich bei dieser Aufgabe meine Schwierigkeiten.
Hoffe ihr könnt mir helfen.

Meine Ideen:
Also die zweite Aufgabe (b) ist ja leicht: elastischer Stoß -> keine bleibende Verformungen und stoßen sich ab.

Bei a muss man ja den Impulssatz vektoriell schreiben, soweit ich weiß:



mehr weiß ich dann auch nicht, kann ehrlich gesagt auch grad nichts mit dem Winkel 30 anfangen.

Gruß
derphysiker40



Anmeldungsdatum: 28.10.2019
Beiträge: 50

Beitrag derphysiker40 Verfasst am: 26. Nov 2019 00:43    Titel: Antworten mit Zitat

Hier die Aufgabe.


physik.png
 Beschreibung:

Download
 Dateiname:  physik.png
 Dateigröße:  105.99 KB
 Heruntergeladen:  147 mal

Myon



Anmeldungsdatum: 04.12.2013
Beiträge: 5864

Beitrag Myon Verfasst am: 26. Nov 2019 11:05    Titel: Re: Stoß zweier Kugeln Antworten mit Zitat

derphysiker40 hat Folgendes geschrieben:
Also die zweite Aufgabe (b) ist ja leicht: elastischer Stoß -> keine bleibende Verformungen und stoßen sich ab.

Das ist nicht richtig. Woraus schliesst Du das, ohne den Aufgabenteil a) gelöst zu haben? Verlangt wäre auch eine Begründung.

Zitat:
Bei a muss man ja den Impulssatz vektoriell schreiben, soweit ich weiß:



Ja das stimmt. Nun kannst Du damit z.B. 2 skalare Gleichungen für die Impulskomponenten parallel / senkrecht zur Anfangsrichtung der beiden Massen aufstellen, denn aus der Vektorgleichung folgt ja, dass der Impuls bezüglich beider Richtungen erhalten sein muss. Damit hat man 2 Gleichungen für die beiden Unbekannten v2n (Geschwindigkeit der 2. Masse nach dem Stoss) sowie deren Streuwinkel .

Erst wenn Du die Geschwindigkeit der 2. Masse nach dem Stoss bestimmt hast, kannst Du prüfen, ob die kinetische Energie erhalten und der Stoss somit elastisch ist.
derphysiker40



Anmeldungsdatum: 28.10.2019
Beiträge: 50

Beitrag derphysiker40 Verfasst am: 26. Nov 2019 11:57    Titel: Antworten mit Zitat

Zitat:
Ja das stimmt. Nun kannst Du damit z.B. 2 skalare Gleichungen für die Impulskomponenten parallel / senkrecht zur Anfangsrichtung der beiden Massen aufstellen, denn aus der Vektorgleichung folgt ja, dass der Impuls bezüglich beider Richtungen erhalten sein muss. Damit hat man 2 Gleichungen für die beiden Unbekannten v2n (Geschwindigkeit der 2. Masse nach dem Stoss) sowie deren Streuwinkel .


Sry, dass ich mich so dumm anstelle, aber ich kann es immer noch nicht ganz nachvollziehen.
Wie stellt man denn sie Skalar Gleichungen auf?
Myon



Anmeldungsdatum: 04.12.2013
Beiträge: 5864

Beitrag Myon Verfasst am: 26. Nov 2019 12:49    Titel: Antworten mit Zitat

Du brauchst Dich für ein Nachfragen überhaupt nicht zu entschuldigen.

Legt man die x-Achse in die Anfangsrichtung der 800g-Masse, so lautet die Vektorgleichung komponentenweise aufgeschrieben



Man hat also die Gleichungen




(mit diesen Vorzeichen ist ). sind bekannt, da der Geschwindigkeitsbetrag und der Winkel gegeben ist. Aus der 1. Gleichung folgt somit , aus der 2. Gleichung .
derphysiker40



Anmeldungsdatum: 28.10.2019
Beiträge: 50

Beitrag derphysiker40 Verfasst am: 26. Nov 2019 13:05    Titel: Antworten mit Zitat

Ok danke erstmal für die nähere Erklärung smile

Ich werde dann versuchen die Aufgabe nachher zumachen (bin grad unterwegs).
Also gegen Abend.

Gruß
derphysiker40



Anmeldungsdatum: 28.10.2019
Beiträge: 50

Beitrag derphysiker40 Verfasst am: 26. Nov 2019 20:37    Titel: Antworten mit Zitat

Nabend,

habe nun alles gemacht und werde es Schritt für Schritt erläutern damit du mir sagen kannst was ich dann falsch gemacht habe, falls da ein Fehler ist.

(alles ohne Vorzeichen damit es schneller geht)

Zu aller erst bestimmt:



Jetzt alles in die Gleichung:




so jetzt nach gelöst ergibt:



Und der Betrag von dem Vektor ergibt dann

Lautet dann die Lösung für 3a) ?

Und wie muss die Begründung für b aussehen?

Gruß
Myon



Anmeldungsdatum: 04.12.2013
Beiträge: 5864

Beitrag Myon Verfasst am: 26. Nov 2019 22:59    Titel: Antworten mit Zitat

Das sieht alles sehr gut aus. Natürlich muss immer noch die Einheit angefügt werden.

Zu b): du brauchst nur die Summe der kinetischen Energien vor und nach dem Stoss auszurechnen. Bleibt die kinetische Energie insgesamt erhalten, handelt es sich um einen elastischen Stoss, nimmt sie ab, ist der Stoss unelastisch (hier wird beim Stoss etwa 74% der kinetischen Energie in andere Energieformen umgewandelt).
derphysiker40



Anmeldungsdatum: 28.10.2019
Beiträge: 50

Beitrag derphysiker40 Verfasst am: 27. Nov 2019 00:23    Titel: Antworten mit Zitat

Alles klar dann und danke für deine Hilfe!

Die vektorielle Schreibweise hat mich anfangs nur wegen den x und y Komponenten verwirrt, aber jetzt war es doch leichter als Gedacht smile

Dir wünsche ich noch eine gute Nacht

Gruß
Neue Frage »
Antworten »
    Foren-Übersicht -> Mechanik